1answer.
Ask question
Login Signup
Ask question
All categories
  • English
  • Mathematics
  • Social Studies
  • Business
  • History
  • Health
  • Geography
  • Biology
  • Physics
  • Chemistry
  • Computers and Technology
  • Arts
  • World Languages
  • Spanish
  • French
  • German
  • Advanced Placement (AP)
  • SAT
  • Medicine
  • Law
  • Engineering
mixer [17]
3 years ago
10

P and q are integers that are multiple of 5. Which of this

Mathematics
1 answer:
zhuklara [117]3 years ago
5 0

Answer:

we conclude that the only option (a) is true.

Step-by-step explanation:

As we know that the multiples of 5 are the numbers which we get when we multiply by 5.

i.e.

5×1=5

5×2=10

Here, 5 and 10 are multiples of 5.

Let p and q are integers that are multiples of 5.

Let us consider

p=5

q=10

so

p+q=5+10

     = 15

A number is divisible by 5 if it ends in 5 or 0.

i.e. 15/5 = 3

so p+q is divisible by 3 as there is no remainder left.

Therefore, option (a) is true.

Checking the other options:

(b) P –q is divisible by 10

As

p=5

q=10

so

p-q=5-10

     = -5

Numbers that are divisible by 10 need to be even or divisible by 2 and divisible by 5.

As -5 is not divisible by 2.

So, option b is NOT true.

(c) P +q is divisible by 20

As

p=5

q=10

so

p-q=5+10

     = 15

Divisibility rule of  20  implies that the last two digits of the number are either  00  or divisible by  20 .

Therefore, P + q= 20 is not divisible by 20 as we don't get the whole number.

(d) P + q is divisible by 25​

As

p=5

q=10

so

p-q=5+10

     = 15

p+q=15 is not divisible by 25 as it does not end with 00, 25, 50, or 75.

so, option d is NOT correct.

Therefore, we conclude that the only option (a) is true.

You might be interested in
....................
frosja888 [35]
Well its pretty obvious that it cant be A  the answer will be E) because abc are connected and abd are as well so the answer will be e
3 0
3 years ago
4. Solve the system of equations.
sweet [91]

The answer should be B (5/3, 0)

4 0
3 years ago
Helppp 80 points no links please
Aneli [31]

Answer:this look really hard to answer i tried everyth

ing

Step-by-step explanation:

4 0
3 years ago
Steel Factory Workers Ages
igor_vitrenko [27]

Solution: We are given the population mean =42

Now, in order to find which shift's mean is closest to population mean, we will find the mean of each shift.

The mean of shift 1 is:

Mean=\frac{18+25+56+42+29+38+54+47+35+30}{10}=\frac{374}{10}=37.4

The mean of shift 2 is:

Mean=\frac{23+19+50+49+67+34+30+59+40+33
}{10}=\frac{404}{10}=40.4

The mean of shift 3 is:

Mean=\frac{19+22+24+40+45+29+33+29+39+59  }{10}=\frac{339}{10}=33.9

The mean of shift 4 is:

Mean=\frac{21+23+25+40+35+19+70+40+22+23  }{10}=\frac{318}{10}=31.8

We clearly see the mean of shift 2 is close to the population mean. Hence the option B) Shift 2 is correct.

7 0
3 years ago
Read 2 more answers
Find the length of AB. A 0140 AB = [?] 140° 8 m B Round your answer to the nearest hundredth.​
lana [24]

\huge{ \mathfrak{  \underline{ Answer} \:  \:  ✓ }}

  • Radius (r) = 8m

  • Angle made at centre (\theta) = 140°

\boxed{ \mathrm{length \:  \: of  \: \: arc =  \dfrac{ \theta}{360 \degree}  \times 2\pi r}}

  • \dfrac{140}{360}  \times 2 \times 3.14 \times 8

  • \dfrac{7}{18}  \times 50.24

  • \dfrac{351.64}{18}

  • 19.54 \:m

_____________________________

\mathrm{ ☠ \: TeeNForeveR \:☠ }

6 0
3 years ago
Other questions:
  • What is the length of the arc when the degree measure is 60° and the radius is 7?
    7·1 answer
  • $700 principle earning 2.25%, compound quarterly, after 6 years
    13·1 answer
  • Explain why any rational number is either a terminating or repeating decimal
    9·1 answer
  • A model rocket is launched from the ground with an initial velocity of 120 feet per second. The height of an object h, in feet,
    13·2 answers
  • The experimental probability of spinning a number greater than 3 is
    10·1 answer
  • What is the best order-of-magnitude for 0.000006?
    9·1 answer
  • Please help I will give brainliest please show work and find the volume of this odd looking pyramid
    7·1 answer
  • Expressions cannot _____.
    9·2 answers
  • Carlo puts tins into small boxes and into large boxes. He puts 6 tins into each small box. He puts 20 tins into each large box.
    15·1 answer
  • A ball is dropped from a helicopter 100m from the ground. As the ball falls, the magnitude of it's associated velocity is
    5·1 answer
Add answer
Login
Not registered? Fast signup
Signup
Login Signup
Ask question!